Limit with arctan: $lim_{xrightarrow 0} frac{xsin 3x}{arctan x^2}$












1












$begingroup$


$$lim_{xrightarrow 0} frac{xsin 3x}{arctan x^2}$$



NB! I haven't learnt about L'Hôpital's rule yet, so I'm still solving limits using common limits.



What I've done so far



$$lim_{xrightarrow0}left[frac{x}{arctan x^2}cdot 3xcdot frac{sin 3x}{3x}right] = 0cdot1cdotlim_{xrightarrow0}left[frac{x}{arctan x^2}right] = 0??$$
Obviously I'm wrong, but I thought I'd show what I tried.










share|cite|improve this question











$endgroup$












  • $begingroup$
    Do you have this inequality: $sin x leq x leq tan x$ for $x geq 0$? And the limits that follow from it: $sin x / x rightarrow 0$, etc.?
    $endgroup$
    – Simon S
    Nov 3 '14 at 23:46


















1












$begingroup$


$$lim_{xrightarrow 0} frac{xsin 3x}{arctan x^2}$$



NB! I haven't learnt about L'Hôpital's rule yet, so I'm still solving limits using common limits.



What I've done so far



$$lim_{xrightarrow0}left[frac{x}{arctan x^2}cdot 3xcdot frac{sin 3x}{3x}right] = 0cdot1cdotlim_{xrightarrow0}left[frac{x}{arctan x^2}right] = 0??$$
Obviously I'm wrong, but I thought I'd show what I tried.










share|cite|improve this question











$endgroup$












  • $begingroup$
    Do you have this inequality: $sin x leq x leq tan x$ for $x geq 0$? And the limits that follow from it: $sin x / x rightarrow 0$, etc.?
    $endgroup$
    – Simon S
    Nov 3 '14 at 23:46
















1












1








1





$begingroup$


$$lim_{xrightarrow 0} frac{xsin 3x}{arctan x^2}$$



NB! I haven't learnt about L'Hôpital's rule yet, so I'm still solving limits using common limits.



What I've done so far



$$lim_{xrightarrow0}left[frac{x}{arctan x^2}cdot 3xcdot frac{sin 3x}{3x}right] = 0cdot1cdotlim_{xrightarrow0}left[frac{x}{arctan x^2}right] = 0??$$
Obviously I'm wrong, but I thought I'd show what I tried.










share|cite|improve this question











$endgroup$




$$lim_{xrightarrow 0} frac{xsin 3x}{arctan x^2}$$



NB! I haven't learnt about L'Hôpital's rule yet, so I'm still solving limits using common limits.



What I've done so far



$$lim_{xrightarrow0}left[frac{x}{arctan x^2}cdot 3xcdot frac{sin 3x}{3x}right] = 0cdot1cdotlim_{xrightarrow0}left[frac{x}{arctan x^2}right] = 0??$$
Obviously I'm wrong, but I thought I'd show what I tried.







calculus limits






share|cite|improve this question















share|cite|improve this question













share|cite|improve this question




share|cite|improve this question








edited Dec 21 '18 at 17:10









Larry

2,53031131




2,53031131










asked Nov 3 '14 at 23:36









B. LeeB. Lee

900916




900916












  • $begingroup$
    Do you have this inequality: $sin x leq x leq tan x$ for $x geq 0$? And the limits that follow from it: $sin x / x rightarrow 0$, etc.?
    $endgroup$
    – Simon S
    Nov 3 '14 at 23:46




















  • $begingroup$
    Do you have this inequality: $sin x leq x leq tan x$ for $x geq 0$? And the limits that follow from it: $sin x / x rightarrow 0$, etc.?
    $endgroup$
    – Simon S
    Nov 3 '14 at 23:46


















$begingroup$
Do you have this inequality: $sin x leq x leq tan x$ for $x geq 0$? And the limits that follow from it: $sin x / x rightarrow 0$, etc.?
$endgroup$
– Simon S
Nov 3 '14 at 23:46






$begingroup$
Do you have this inequality: $sin x leq x leq tan x$ for $x geq 0$? And the limits that follow from it: $sin x / x rightarrow 0$, etc.?
$endgroup$
– Simon S
Nov 3 '14 at 23:46












4 Answers
4






active

oldest

votes


















2












$begingroup$

You should put in
$$
frac{x^2}{arctan x^2}
$$
that has limit $1$:
$$
lim_{xto0}frac{xsin3x}{arctan x^2}=
lim_{xto0}3frac{sin3x}{3x}frac{x^2}{arctan x^2}=dots
$$
If you don't know the limit above, just substitute $t=arctan x^2$, so $x^2=tan t$ and the limit is
$$
lim_{xto0}frac{x^2}{arctan x^2}=lim_{tto0}frac{tan t}{t}
$$
that you should be able to manage.






share|cite|improve this answer









$endgroup$





















    4












    $begingroup$

    Starting as you did, we want to find the limit of
    $$frac{3x^2}{arctan(x^2)}cdotfrac{sin(3x)}{3x}.$$
    Only the first term gives any trouble. Let $x$ be not too large, and let $x^2=tan w$, You want to find
    $$lim_{wto 0} frac{3tan w}{w},$$
    which is not difficult. Replace $tan w$ by $frac{sin w}{cos w}$.






    share|cite|improve this answer









    $endgroup$





















      1












      $begingroup$

      Since $u sim tan u$, then $y sim arctan y$, so $x^2 sim arctan (x^2)$.



      This means that
      $$lim_{x to 0} frac{x sin 3x}{arctan x^2} = lim_{x to 0} frac{x3x}{x^2} = 3$$






      share|cite|improve this answer









      $endgroup$













      • $begingroup$
        This feels unsatisfactory, as you are still using the derivatives of the functions in the numerator and denominator.
        $endgroup$
        – Simon S
        Nov 3 '14 at 23:42



















      0












      $begingroup$

      Using the Asymptotic expansion:

      $sin(x)approx_0 x$

      $arctan(x^2) approx_0 x^2$

      So
      $$lim _{xrightarrow :0}:frac{xsin :3x}{arctan :x^2}=lim :_{xrightarrow 0}:frac{3x^2}{x^2}=color{red}{3}$$






      share|cite|improve this answer









      $endgroup$













        Your Answer





        StackExchange.ifUsing("editor", function () {
        return StackExchange.using("mathjaxEditing", function () {
        StackExchange.MarkdownEditor.creationCallbacks.add(function (editor, postfix) {
        StackExchange.mathjaxEditing.prepareWmdForMathJax(editor, postfix, [["$", "$"], ["\\(","\\)"]]);
        });
        });
        }, "mathjax-editing");

        StackExchange.ready(function() {
        var channelOptions = {
        tags: "".split(" "),
        id: "69"
        };
        initTagRenderer("".split(" "), "".split(" "), channelOptions);

        StackExchange.using("externalEditor", function() {
        // Have to fire editor after snippets, if snippets enabled
        if (StackExchange.settings.snippets.snippetsEnabled) {
        StackExchange.using("snippets", function() {
        createEditor();
        });
        }
        else {
        createEditor();
        }
        });

        function createEditor() {
        StackExchange.prepareEditor({
        heartbeatType: 'answer',
        autoActivateHeartbeat: false,
        convertImagesToLinks: true,
        noModals: true,
        showLowRepImageUploadWarning: true,
        reputationToPostImages: 10,
        bindNavPrevention: true,
        postfix: "",
        imageUploader: {
        brandingHtml: "Powered by u003ca class="icon-imgur-white" href="https://imgur.com/"u003eu003c/au003e",
        contentPolicyHtml: "User contributions licensed under u003ca href="https://creativecommons.org/licenses/by-sa/3.0/"u003ecc by-sa 3.0 with attribution requiredu003c/au003e u003ca href="https://stackoverflow.com/legal/content-policy"u003e(content policy)u003c/au003e",
        allowUrls: true
        },
        noCode: true, onDemand: true,
        discardSelector: ".discard-answer"
        ,immediatelyShowMarkdownHelp:true
        });


        }
        });














        draft saved

        draft discarded


















        StackExchange.ready(
        function () {
        StackExchange.openid.initPostLogin('.new-post-login', 'https%3a%2f%2fmath.stackexchange.com%2fquestions%2f1005029%2flimit-with-arctan-lim-x-rightarrow-0-fracx-sin-3x-arctan-x2%23new-answer', 'question_page');
        }
        );

        Post as a guest















        Required, but never shown

























        4 Answers
        4






        active

        oldest

        votes








        4 Answers
        4






        active

        oldest

        votes









        active

        oldest

        votes






        active

        oldest

        votes









        2












        $begingroup$

        You should put in
        $$
        frac{x^2}{arctan x^2}
        $$
        that has limit $1$:
        $$
        lim_{xto0}frac{xsin3x}{arctan x^2}=
        lim_{xto0}3frac{sin3x}{3x}frac{x^2}{arctan x^2}=dots
        $$
        If you don't know the limit above, just substitute $t=arctan x^2$, so $x^2=tan t$ and the limit is
        $$
        lim_{xto0}frac{x^2}{arctan x^2}=lim_{tto0}frac{tan t}{t}
        $$
        that you should be able to manage.






        share|cite|improve this answer









        $endgroup$


















          2












          $begingroup$

          You should put in
          $$
          frac{x^2}{arctan x^2}
          $$
          that has limit $1$:
          $$
          lim_{xto0}frac{xsin3x}{arctan x^2}=
          lim_{xto0}3frac{sin3x}{3x}frac{x^2}{arctan x^2}=dots
          $$
          If you don't know the limit above, just substitute $t=arctan x^2$, so $x^2=tan t$ and the limit is
          $$
          lim_{xto0}frac{x^2}{arctan x^2}=lim_{tto0}frac{tan t}{t}
          $$
          that you should be able to manage.






          share|cite|improve this answer









          $endgroup$
















            2












            2








            2





            $begingroup$

            You should put in
            $$
            frac{x^2}{arctan x^2}
            $$
            that has limit $1$:
            $$
            lim_{xto0}frac{xsin3x}{arctan x^2}=
            lim_{xto0}3frac{sin3x}{3x}frac{x^2}{arctan x^2}=dots
            $$
            If you don't know the limit above, just substitute $t=arctan x^2$, so $x^2=tan t$ and the limit is
            $$
            lim_{xto0}frac{x^2}{arctan x^2}=lim_{tto0}frac{tan t}{t}
            $$
            that you should be able to manage.






            share|cite|improve this answer









            $endgroup$



            You should put in
            $$
            frac{x^2}{arctan x^2}
            $$
            that has limit $1$:
            $$
            lim_{xto0}frac{xsin3x}{arctan x^2}=
            lim_{xto0}3frac{sin3x}{3x}frac{x^2}{arctan x^2}=dots
            $$
            If you don't know the limit above, just substitute $t=arctan x^2$, so $x^2=tan t$ and the limit is
            $$
            lim_{xto0}frac{x^2}{arctan x^2}=lim_{tto0}frac{tan t}{t}
            $$
            that you should be able to manage.







            share|cite|improve this answer












            share|cite|improve this answer



            share|cite|improve this answer










            answered Nov 3 '14 at 23:47









            egregegreg

            185k1486206




            185k1486206























                4












                $begingroup$

                Starting as you did, we want to find the limit of
                $$frac{3x^2}{arctan(x^2)}cdotfrac{sin(3x)}{3x}.$$
                Only the first term gives any trouble. Let $x$ be not too large, and let $x^2=tan w$, You want to find
                $$lim_{wto 0} frac{3tan w}{w},$$
                which is not difficult. Replace $tan w$ by $frac{sin w}{cos w}$.






                share|cite|improve this answer









                $endgroup$


















                  4












                  $begingroup$

                  Starting as you did, we want to find the limit of
                  $$frac{3x^2}{arctan(x^2)}cdotfrac{sin(3x)}{3x}.$$
                  Only the first term gives any trouble. Let $x$ be not too large, and let $x^2=tan w$, You want to find
                  $$lim_{wto 0} frac{3tan w}{w},$$
                  which is not difficult. Replace $tan w$ by $frac{sin w}{cos w}$.






                  share|cite|improve this answer









                  $endgroup$
















                    4












                    4








                    4





                    $begingroup$

                    Starting as you did, we want to find the limit of
                    $$frac{3x^2}{arctan(x^2)}cdotfrac{sin(3x)}{3x}.$$
                    Only the first term gives any trouble. Let $x$ be not too large, and let $x^2=tan w$, You want to find
                    $$lim_{wto 0} frac{3tan w}{w},$$
                    which is not difficult. Replace $tan w$ by $frac{sin w}{cos w}$.






                    share|cite|improve this answer









                    $endgroup$



                    Starting as you did, we want to find the limit of
                    $$frac{3x^2}{arctan(x^2)}cdotfrac{sin(3x)}{3x}.$$
                    Only the first term gives any trouble. Let $x$ be not too large, and let $x^2=tan w$, You want to find
                    $$lim_{wto 0} frac{3tan w}{w},$$
                    which is not difficult. Replace $tan w$ by $frac{sin w}{cos w}$.







                    share|cite|improve this answer












                    share|cite|improve this answer



                    share|cite|improve this answer










                    answered Nov 3 '14 at 23:50









                    André NicolasAndré Nicolas

                    454k36432819




                    454k36432819























                        1












                        $begingroup$

                        Since $u sim tan u$, then $y sim arctan y$, so $x^2 sim arctan (x^2)$.



                        This means that
                        $$lim_{x to 0} frac{x sin 3x}{arctan x^2} = lim_{x to 0} frac{x3x}{x^2} = 3$$






                        share|cite|improve this answer









                        $endgroup$













                        • $begingroup$
                          This feels unsatisfactory, as you are still using the derivatives of the functions in the numerator and denominator.
                          $endgroup$
                          – Simon S
                          Nov 3 '14 at 23:42
















                        1












                        $begingroup$

                        Since $u sim tan u$, then $y sim arctan y$, so $x^2 sim arctan (x^2)$.



                        This means that
                        $$lim_{x to 0} frac{x sin 3x}{arctan x^2} = lim_{x to 0} frac{x3x}{x^2} = 3$$






                        share|cite|improve this answer









                        $endgroup$













                        • $begingroup$
                          This feels unsatisfactory, as you are still using the derivatives of the functions in the numerator and denominator.
                          $endgroup$
                          – Simon S
                          Nov 3 '14 at 23:42














                        1












                        1








                        1





                        $begingroup$

                        Since $u sim tan u$, then $y sim arctan y$, so $x^2 sim arctan (x^2)$.



                        This means that
                        $$lim_{x to 0} frac{x sin 3x}{arctan x^2} = lim_{x to 0} frac{x3x}{x^2} = 3$$






                        share|cite|improve this answer









                        $endgroup$



                        Since $u sim tan u$, then $y sim arctan y$, so $x^2 sim arctan (x^2)$.



                        This means that
                        $$lim_{x to 0} frac{x sin 3x}{arctan x^2} = lim_{x to 0} frac{x3x}{x^2} = 3$$







                        share|cite|improve this answer












                        share|cite|improve this answer



                        share|cite|improve this answer










                        answered Nov 3 '14 at 23:41









                        CrostulCrostul

                        28.2k22352




                        28.2k22352












                        • $begingroup$
                          This feels unsatisfactory, as you are still using the derivatives of the functions in the numerator and denominator.
                          $endgroup$
                          – Simon S
                          Nov 3 '14 at 23:42


















                        • $begingroup$
                          This feels unsatisfactory, as you are still using the derivatives of the functions in the numerator and denominator.
                          $endgroup$
                          – Simon S
                          Nov 3 '14 at 23:42
















                        $begingroup$
                        This feels unsatisfactory, as you are still using the derivatives of the functions in the numerator and denominator.
                        $endgroup$
                        – Simon S
                        Nov 3 '14 at 23:42




                        $begingroup$
                        This feels unsatisfactory, as you are still using the derivatives of the functions in the numerator and denominator.
                        $endgroup$
                        – Simon S
                        Nov 3 '14 at 23:42











                        0












                        $begingroup$

                        Using the Asymptotic expansion:

                        $sin(x)approx_0 x$

                        $arctan(x^2) approx_0 x^2$

                        So
                        $$lim _{xrightarrow :0}:frac{xsin :3x}{arctan :x^2}=lim :_{xrightarrow 0}:frac{3x^2}{x^2}=color{red}{3}$$






                        share|cite|improve this answer









                        $endgroup$


















                          0












                          $begingroup$

                          Using the Asymptotic expansion:

                          $sin(x)approx_0 x$

                          $arctan(x^2) approx_0 x^2$

                          So
                          $$lim _{xrightarrow :0}:frac{xsin :3x}{arctan :x^2}=lim :_{xrightarrow 0}:frac{3x^2}{x^2}=color{red}{3}$$






                          share|cite|improve this answer









                          $endgroup$
















                            0












                            0








                            0





                            $begingroup$

                            Using the Asymptotic expansion:

                            $sin(x)approx_0 x$

                            $arctan(x^2) approx_0 x^2$

                            So
                            $$lim _{xrightarrow :0}:frac{xsin :3x}{arctan :x^2}=lim :_{xrightarrow 0}:frac{3x^2}{x^2}=color{red}{3}$$






                            share|cite|improve this answer









                            $endgroup$



                            Using the Asymptotic expansion:

                            $sin(x)approx_0 x$

                            $arctan(x^2) approx_0 x^2$

                            So
                            $$lim _{xrightarrow :0}:frac{xsin :3x}{arctan :x^2}=lim :_{xrightarrow 0}:frac{3x^2}{x^2}=color{red}{3}$$







                            share|cite|improve this answer












                            share|cite|improve this answer



                            share|cite|improve this answer










                            answered Oct 27 '16 at 20:49









                            AmarildoAmarildo

                            1,789816




                            1,789816






























                                draft saved

                                draft discarded




















































                                Thanks for contributing an answer to Mathematics Stack Exchange!


                                • Please be sure to answer the question. Provide details and share your research!

                                But avoid



                                • Asking for help, clarification, or responding to other answers.

                                • Making statements based on opinion; back them up with references or personal experience.


                                Use MathJax to format equations. MathJax reference.


                                To learn more, see our tips on writing great answers.




                                draft saved


                                draft discarded














                                StackExchange.ready(
                                function () {
                                StackExchange.openid.initPostLogin('.new-post-login', 'https%3a%2f%2fmath.stackexchange.com%2fquestions%2f1005029%2flimit-with-arctan-lim-x-rightarrow-0-fracx-sin-3x-arctan-x2%23new-answer', 'question_page');
                                }
                                );

                                Post as a guest















                                Required, but never shown





















































                                Required, but never shown














                                Required, but never shown












                                Required, but never shown







                                Required, but never shown

































                                Required, but never shown














                                Required, but never shown












                                Required, but never shown







                                Required, but never shown







                                Popular posts from this blog

                                Bundesstraße 106

                                Le Mesnil-Réaume

                                Ida-Boy-Ed-Garten